PT20.S1.Q14 - yolanda: gaining access to computers

TheoryandPracticeTheoryandPractice Alum Member
edited March 2017 in Logical Reasoning 1008 karma

Why is the correct answer C? I did watch JY's video, but still not sure why B is wrong. Here's my reasoning:

Yolanda’s conclusion: Joyriding is the MORE dangerous crime (than gaining access to computers without authorization)
Arjun’s reasoning: I disagree (or meaning, Joyriding is NOT MORE dangerous crime (than gaining access to computers without authorization)
Why? Because computer crimes also cause physical harm to people.

Here I think the Arjun’s flaw is that what’s absolute (computer crimes cause harm) cannot prove what's relative (computer crimes cause MORE harm than joyriding.
Yolanda’s making a comparison argument (joyriding is MORE dangerous) but Arjun’s disagreeing with it with only the evidence that computer crime also causes harm. He does not establish that computer crime is MORE dangerous, only that it is dangerous.

That’s why I thought that he does not provide a valid evidence to disagree with Yolanda. Am I reading answer choice B wrong? Would any evidence, albeit an invalid one, be fine to eliminate answer choice B?
https://7sage.com/lsat_explanations/lsat-20-section-1-question-14/

Comments

  • SamiSami Live Member Sage 7Sage Tutor
    10774 karma

    @TheoryandPractice said:

    Here I think the Arjun’s flaw is that what’s absolute (computer crimes cause harm) cannot prove what's relative (computer crimes cause MORE harm than joyriding.
    Yolanda’s making a comparison argument (joyriding is MORE dangerous) but Arjun’s disagreeing with it with only the evidence that computer crime also causes harm. He does not establish that computer crime is MORE dangerous, only that it is dangerous.

    Hey so take a look at Yolanda's argument. when she compares computer crimes to joyriding in stolen cars she says "both involve ..." .
    -Which is pretty absolute. They both have similar characteristics.

    But Arjun does it by saying "that it coulddamage". Therefore he concludes saying computer crimes also cause physical harm to people. Arjun is going from what could be true in one instance to what is absolutely going to cause that affect! That's the flaw!

    So answer choice "C" is pinpointing to that. Arjun relies on the actuality of a phenomena, computer crimes causing physical harm, to what he has only shown to be possible "could damage" in his premise.

    Am I reading answer choice B wrong? Would any evidence, albeit an invalid one, be fine to eliminate answer choice B?

    Answer choice "B" is saying Arjun doesn't provide any evidence for denying Yolanda's conclusion. But Arjun does provide 1 fact/evidence to back up his conclusion that denies Yolanda's conclusion. That fact is that unauthorized use of medical records could damage data systems on which human lives depend. So answer choice "B" is just factually wrong in that its stating that Arjun did not do something when he in fact did do that.

    I hope this helps. : )

  • TheoryandPracticeTheoryandPractice Alum Member
    1008 karma

    Thanks a lot everyone!

  • zoehlillianzoehlillian Free Trial Member
    2 karma

    Going to bring this back up. Sorry!

    Hey so take a look at Yolanda's argument. when she compares computer crimes to joyriding in stolen cars she says "both involve ..." .
    -Which is pretty absolute. They both have similar characteristics.

    But Isn't Yolanda's conclusion that joyriding is MORE dangerous than computer crimes? If so, then doesn't Arjun fail to provide evidence that the relative is false, by simply saying it is also (albeit only potentially) dangerous? (Arjun doesn't give any evidence that joyriding is LESS dangerous than computer crimes, only posits that computer crimes could be dangerous. Or do I have the conclusion wrong?)

Sign In or Register to comment.